Vous êtes sur la page 1sur 4

Math 206, Spring 2016

Assignment 7 Solutions

Due: March 11, 2016

Part A.
(1) Suppose that T L(Rc , Rr ) and S L(Rr , Rq ). Prove that ker(T ) ker(S T ), and that im(S T )
im(S).
Solution. Well first prove that ker(T ) ker(S T ). For this, let x ker(T ) be given. This means
that T (x) = 0. Note further that since S is linear, we have that S(0) = S(00) = 0S(0) = 0, where the
final equality is because any vector scaled by 0 is 0. Hence we must have
(S T )(x) = S(T (x))

(definition of composition)
(since x ker(T ))

= S(0)
=0

(since S(0) = 0).

Hence we have that x ker(S T ).


Now we prove that Im(S T ) Im(S). For this, let b im(S T ) be given. This means that
there exists some x Rc so that (S T )(x) = b. But the definition of composition then tells us that
S(T (x)) = b, and hence b is the output (under the function S) of the element T (x) Rr . Hence
b im(S), as desired.

(2) Suppose that T L(Rc , Rr ) is injective, and suppose that {v1 , , vk } Rc is a linearly independent
collection. Prove that {T (v1 ), , T (vk )} is linearly independent.
Solution. To show that {T (v1 ), , T (vk )} is independent, let c Rk be some linear relation. In
other words, suppose that
c1 T (v1 ) + + ck T (vk ) = 0.
If we can show that c = 0, then we will have shown the set is independent.
Notice that the linearity of T turns our given relation into the following equation
T (c1 v1 + + ck vk ) = 0.
Of course, we also know that T (0) = 0 [see, for instance, the proof in the previous problem, or the
argument we gave in class]. But since T is injective, the equality
T (c1 v1 + + ck vk ) = T (0)
forces the equality
c1 v1 + + ck vk = 0.
In other words, we have shown that c is a linear relation of {v1 , , vk } as well. Since were told that
{v1 , , vk } is independent, we conclude that c = 0, as desired.

(3) (?) Your friend has just heard that a subspace W Rn is a set which satisfies: (1) 0 W ; (2) if
w1 , w2 W , then w1 + w2 W ; and (3) if w W and k R, then kw W . [This is the definition we
gave in class.] She tries to convince you that the first condition is unnecessary and should be omitted
so that the definition of a subspace is simpler and more elegant. Heres her argument:
We know that W is closed under scaling. So if you pick up any w W and scale it by 0,
then the result has to be in W . But we already said in class that if you scale anything by 0,
the result is 0. Hence condition (1) is just a consequence of condition (3).
http://palmer.wellesley.edu/~aschultz/w16/math206

Page 1 of 4

Math 206, Spring 2016

Assignment 7 Solutions

Due: March 11, 2016

Explain to your friend why her argument isnt quite right.


Solution. Her argument breaks down precisely in the case where W is the empty set. In this case
the procedure pick up any w W cant be completed, so theres no vector to scale which will allow
you to produce 0. Other than this her argument works perfectly well. [And, in fact, one could replace
condition (1) from class with the seemingly more tame assertion that W 6= .]

Part B.
(1) Suppose that {v, z, w} is a basis for some subspace W in Rn . Prove that {v + w, w + z, v + z} is also
a basis for W .
Solution. We need to verify that {v + w, w + z, v + z} is linearly independent and that they span
W . Well resolve independence first. Suppose that c R3 is some relation amongst these vectors:
c1 (v + w) + c2 (w + z) + c3 (v + z) = 0.
We will prove the set is independent by showing that c must, in fact, be 0. To do this, note that our
equation is equivalent (after distributing and rearranging terms) to
(c1 + c3 )v + (c1 + c2 )w + (c2 + c3 )z = 0.
Since we know that {v, w, z} is a basis, it must be independent, and hence we must have c1 + c3 =
c1 + c2 = c2 + c3 = 0. But this is system of equations which we can solve via row reduction:

1 0
1 0
1 0
1 0
1 0 1 0
1 +2
2 +3
1 1 0 0 0 1 1 0 0 1 1 0
0 1 1 0
0 1
1 0
0 0
2 0

1 0
1
(1/2)3
0 1 1
0 0
1

0
1 0 1
3 +2
0 0 1 0
0 0 1
0

0
1 0 0
3 +1
0 0 1 0
0 0 1
0

0
0 .
0

Hence we get c1 = c2 = c3 = 0, as desired.


Now well show that {v + w, w + z, v + z} spans W . Let x W be given; our goal is to find
d1 , d2 , d3 R so that
x = d1 (v + w) + d2 (w + z) + d3 (v + z).
Recall that were told {v, w, z} is a basis for W , and hence must be a spanning set. Therefore we do
know that there are constants b1 , b2 , b3 R so that
x = b1 v + b2 w + b3 z.
With this in mind, observe that
b1 + b2 b3
b1 + b2 + b3
b1 b2 + b3
(v + w) +
(w + z) +
(v + z)
2
2
2






b1 + b2 b3
b1 b2 + b3
b1 + b2 b3
b1 + b2 + b3
b1 + b2 + b3
b1 b2 + b3
=
+
v+
+
w+
+
z
2
2
2
2
2
2
= b1 v + b2 w + b3 z = x.
Hence x is a linear combination of {v + w, w + z, v + z}, and this set spans W as desired.

[Note: in the proof of that the given set spans above, it may seem that we pulled some pretty
sophisticated constants out of thin air which magically produced our desired vector. However, this
is only because arguing that a certain vector is in the span of a collection amounts to verifying some
http://palmer.wellesley.edu/~aschultz/w16/math206

Page 2 of 4

Math 206, Spring 2016

Assignment 7 Solutions

Due: March 11, 2016

existential statement. As weve seen before, the work in proving existential statements often comes from
doing some hard work on a scratch piece of paper to find the quantities you claim exist. In this case,
we knew that there were constants b1 , b2 , b3 R so that
x = b1 v + b2 w + b3 z,
and we wanted constants d1 , d2 , d3 R so that
x = d1 (v + w) + d2 (w + z) + d3 (v + z).
If we take that last statement and manipulate it (something we cant do in a proof, since that would be
manipulating an equation we want to be true!), it would yield
x = (d1 + d3 )v + (d1 + d2 )w + (d2 + d3 )z.
Hence to find d1 , d2 and d3 which satisfy the equation we want to be true, we really need to solve the
system

d1 + d3 = b1
d1 + d2 = b2
.

d2 + d3 = b3
We produced the constants in our proof by simply solving this linear system.]
(2) Suppose that V and W are subspaces in Rn , and define
V + W = {v + w : v V, w W }.
Prove that V + W is a subspace of Rn .
Solution. We need to verify the three axioms of a subspace.
First, we must show that 0 V + W . For this, recall that since V and W are subspaces, we have
0 V and 0 W . Hence we have that 0 + 0 V + W . But of course 0 + 0 = 0, so we have shown
0 V + W as desired.
Now we will show that V + W is closed under addition. Let z1 , z2 V + W be given. To say that
z1 V + W means there exist v1 V and w1 W so that z1 = v1 + w1 . Likewise there exist v2 V
and w2 W with z2 = v2 + w2 . We then get
z1 + z2 = (v1 + w1 ) + (v2 + w2 ) = (v1 + v2 ) + (w1 + w2 ).
But observe that since V is closed under addition, we have v1 + v2 V ; likewise since W is closed
under addition, we get w1 + w2 W . Hence we have exhibited z1 + z2 as the sum of a vector from V
and a vector from W , and therefore is in V + W .
Finally, we show that V + W is closed under scaling. let z V + W and k R be given. By the
definition of V + W , there exist some v V and w W with z = v + w. Note that
kz = k(v + w) = kv + kw.
Since V is closed under scaling, we have kv V ; likewise since W is closed under scaling, we have
kw W . So we have exhibited kz as a sum of a vector from V and a vector from W , and hence
kz V + W .

(3) Suppose that W is a subspace of Rr , and that T L(Rc , Rr ). Prove that
T 1 (W ) = {w Rc : T (w) W }
is a subspace of Rc .br[Note: One reads T 1 (W ) as the preimage of W under T . This notation
is defined even when T is not a bijection, because were not attempting to define some function T 1 :
Rr Rc . Instead, T 1 (W ) is simply meant to represent the set of al inputs of T whose corresponding
output is in W . In particular, in the special case where W = {0}, note that T 1 ({0}) is just ker(T ).]
http://palmer.wellesley.edu/~aschultz/w16/math206

Page 3 of 4

Math 206, Spring 2016

Assignment 7 Solutions

Due: March 11, 2016

Solution. Well verify the three axioms of a subspace.


[For this first axiom, Ill use the notation 0c to denote the zero vector in Rc , and 0r to denote the zero
vector in Rr . This will just help alleviate any confusion about where particular elements live.] First,
we must argue that 0c T 1 (W ). For this, note that since T is linear we know that T (0c ) = 0r .
Furthermore we know that 0r W since W is a subspace of Rr . Hence we have T (0c ) W , which is
what it means to say that 0c T 1 (W ).
Now well argue that T 1 (W ) is closed under addition. Let v1 , v2 T 1 (W ) be given. This means
that T (v1 ), T (v2 ) W . Since W is closed under addition, this means that T (v1 ) + T (v2 ) W . The
linearity of T tells us that T (v1 + v2 ) = T (v1 ) + T (v2 ), and hence we have T (v1 + v2 ) W . But this
is what it means to say that v1 + v2 T 1 (W ), so T 1 (W ) is closed under addition.
Now well argue that T 1 (W ) is closed under scaling. Let v T 1 (W ) and k R be given. This
means that T (v) W . Since W is closed under scaling, this means that kT (v) W . The linearity of
T tells us that kT (v) = T (kv), and hence we have T (kv) W . But this is what it means to say that
kv T 1 (W ), so T 1 (W ) is closed under scaling.


http://palmer.wellesley.edu/~aschultz/w16/math206

Page 4 of 4

Vous aimerez peut-être aussi